LSAT and Law School Admissions Forum

Get expert LSAT preparation and law school admissions advice from PowerScore Test Preparation.

User avatar
 Dave Killoran
PowerScore Staff
  • PowerScore Staff
  • Posts: 5853
  • Joined: Mar 25, 2011
|
#87698
Setup and Rule Diagram Explanation

This is a Basic Linear: Balanced, Identify the Templates game.

After reviewing the game scenario, you should make the following basic setup for this game:

PT57-Jun2009_LGE-G1_srd1.png

The first two rules create a block-sequence:

PT57-Jun2009_LGE-G1_srd2.png

This combination results in six Not Laws:
PT57-Jun2009_LGE-G1_srd3.png
The third and fourth rules create another block-sequence, although the block in this sequence is not fixed:

PT57-Jun2009_LGE-G1_srd4.png

Because the block rotates, only four additional Not Laws result:
PT57-Jun2009_LGE-G1_srd5.png

The first and last activities are the most restricted. Only J, K, or M could be the first activity, and only G or L could be the last activity.

Note that the interaction of the two blocks is critical, and leads to a number of restrictions. Thus, this is a perfect scenario to Identify the Templates. Because the HG block is fixed, use the HG block and examine the four templates that initially appear possible:

PT57-Jun2009_LGE-G1_srd6.png
Thus, only three viable templates exists with the HG block (a similar situation occurs with the JM block). Note some of the interesting inferences that the templates reveal:

Only H, G, J, and M can be second, and thus neither K nor L is second. This information answers question #3.
Only H, G, J, and M can be fifth, and thus neither K nor L is fifth. This information answers question #4.
Combining all of this information results in the final setup for the game:

PT57-Jun2009_LGE-G1_srd7.png
You do not have the required permissions to view the files attached to this post.
 Brittney
  • Posts: 5
  • Joined: Oct 16, 2013
|
#11989
I know this is a basic linear game and the setup should be really easy but I think I'm missing a few not laws. I know L, H, G can't be first. G can't be second or fifth. J, M, H, K can't be last. What am I missing?
User avatar
 Dave Killoran
PowerScore Staff
  • PowerScore Staff
  • Posts: 5853
  • Joined: Mar 25, 2011
|
#11991
Hi Brittney,

There's actually quite a bit going on in this game. As I note in our explanation to that game, "the interaction of the two blocks is critical, and leads to a number of restrictions. While placing the blocks at the “ends” of the diagram (JM early and HG late) does not tend to create many problems, as either of the blocks (or their connecting variables) are moved into the “middle” of the diagram, problems begin to result."

What ultimately occurs is that you get more "interior" Not Laws, such as that G, L, and K can't be second, H can't be third, G, can't be fourth, and K and L can't be fifth. Those Not Laws then destroy questions #2, #3, and #4.

Please let me know if that helps. Thanks!
 Dajpol
  • Posts: 14
  • Joined: Jun 23, 2016
|
#28323
Hi Dave,

The hyperlink you included in your response does not work. Could you fix the issue? Interested to see the content you reference.

Thanks!
User avatar
 Dave Killoran
PowerScore Staff
  • PowerScore Staff
  • Posts: 5853
  • Joined: Mar 25, 2011
|
#28340
Hi Dajpol,

As you might have noticed, that original discussion is from 2013. That link, which I have now removed, used to be live but is no longer active and it went to our PDF Download Store. You used to be able to download LSATs and explanations for a minimal fee from that store, and it was really convenient. Then, a few years ago, LSAC changed the licensing rules and in the process made it very difficult for companies to publicly sell PDFs (let's put it this way: you can still sell them, but the costs involved make it challenging at $8 a test). So, there is no explanation file I can link you to you any longer.

We are currently in the process of uploading written explanations for all questions, and free video explanations for every Logic Game (see http://blog.powerscore.com/lsat/announc ... deo-series for more information) but because there are so many questions, this process is taking some time. So, we will add an explanation for this game, but I can't tell you the exact date. In the meantime, if you have a specific questions about the rules or setup of this one, just post it in and we'll get to it as soon as we can.

Thanks!
 vgm789
  • Posts: 2
  • Joined: Aug 06, 2019
|
#67338
Hello,

Can someone please explain the setup and key inferences for this game? As I was going through the questions for this game, specifically questions 2-4, I found that I hadn't made any of these inferences after I set up my diagram. Any help would be much appreciated. Thanks!
 James Finch
PowerScore Staff
  • PowerScore Staff
  • Posts: 943
  • Joined: Sep 06, 2017
|
#67341
Hi VGM,

The inferences needed for this game are clearest if you have a diagram that shows the two variable sequence chains that the rules give us, preferably one on top of the other. The one I created was:

K--[HG]
[M/J]--L

This allows us to represent the two blocks that can't be split and allows us to mentally fit the two chains together in various ways, basically playing Tetris in our heads. For example, you can see from this diagram that K can either go before or after the MJ block and before or after the L, meaning it can go in 1st, 3rd or 4th, answering question 3 for us. This also reminds us that we can't split blocks by putting one of the K or L lone variables in 2nd or 5th. The HG block works similarly, as it must move around the MJ block. But while the M and J can flip positions, H and G are set, meaning that H is forced into either the 2nd, 4th or 5th slot. This then gives us the answer to question 2. Question 4 is asking about 5th, which is the second to last position, meaning it must have one of the blocks in it (otherwise it would be split, as noted above).

Hope this clears things up!

Get the most out of your LSAT Prep Plus subscription.

Analyze and track your performance with our Testing and Analytics Package.